entradas recentes
From categories:
page 1123...next »

Olá,

Penso que há alguma confusão na sua formulação. Pelo que percebi sobre aquilo que esperava que desse, penso que a sua ideia seria calcular o comprimento do gráfico do seno no intervalo $[0,2\pi]$.

Se é essa a questão, a parametrização que interessa usar é $\, t \mapsto (t,\sin t)\,$ no referido intervalo. A parametrição que usou percorreu o segmento $[0,1]$ da esquerda para a direita, depois da direita para a esquerda, e a seguir percorreu o segmento $[-1,0]$ da direita para a esquerda e depois da esquerda para a direita (é a variação do seno em $[0,2\pi]$), e por isso é que dá 4.

Usando a parametrização correta, o integral a calcular é

(1)
\begin{align} \int_0^{2\pi} \sqrt{1+\cos^2t} \; dt. \end{align}

Não creio que seja possível calculá-lo pelas regras habituais de primitivação, mas o seguinte valor aproximado pode ser obtido (por exemplo usando a WolframAlpha): $7,6404$.

Espero ter ajudado!

Re: Sen(t) by acaetanoacaetano, 12 Apr 2014 20:10

Boa noite!

Estive tentando resolver uma questão e não consigo sair de um impasse…

A questão é sobre a função f(t):[0,1]->R ; f(t)=sin(t).

Como o seno é continuamente diferenciável (já que o cosseno é contínuo), suponho que poderia fazer:

(1)
\begin{align} \int_{0}^{2\pi} \left \| {sin(t)}' \right \| dt = \int_{0}^{2\pi} \left \| cos(t) \right \| dt = 4 \times \int_{0}^{\pi/2} cos(t) dt = 4 \times sin(t) dt [0,\pi/2] = 4 \times 1 = 4 \end{align}

Eu errei algo?

Bem, se esta conta estiver certa, se nós prestarmos atenção no gráfico, vamos ver que o valor deve ser maior que 4, pois se fosse em linha reta, seria 2pi… (aproximadamente 6)…

Então, você poderia me ajudar? Onde eu errei?

Sen(t) by Suzugamori HimeSuzugamori Hime, 11 Apr 2014 21:35

Pelo teu exemplo, o que queres é o processo de cálculo do plano tangente a um ponto de uma superfície de nível. Vê http://amiii.wikidot.com/3-1-derivadas-e-gradientes-parte-6#toc0.

Re: R^3 by acaetanoacaetano, 03 Feb 2012 23:33

Como é que se calcula o plano tangente de uma função em R3?
Por exemplo x2+y2+z2=6

R^3 by Ian ChongIan Chong, 01 Feb 2012 22:54

Trata-se do ex. 3.(b) da Folha 1 deste sítio.

Penso que o que queres dizer é que é mais fácil obter a equação na forma de $x$ em função de $y$ (e nesse caso diz-se que o $x$ depende de $y$ — em suma, esta é que é mais fácil). Mas repara que isto deverá ser suficiente para resolver o problema do esboço: é só uma questão de olhares para o sistema de eixos coordenados do ponto de vista adequado.

De qualquer modo, se quiseres obter uma equação com $y$ a depender de $x$, começa por resolver $x=t^2+2t$ em ordem a $t$ como se se tratasse de um equação do 2º grau em $t$ (logo interpretando momentaneamente o $x$ como constante e usando a fórmula resolvente).

Tenho uma duvida. Estou a tentar definir uma possivel equação cartesiana e esboçar a mesma a partir das seguintes paramétricas:

r(t) = (t^2 + 2t; t^4 + 1); t € [0; 1]

O meu problema é o seguinte, não consigo criar uma equação com o x dependente de y. Só o contrário. Será que me pude ajudar a resolver esta questão?

As notas do exame de recurso serão afixadas nesta linha (thread) na 3ª-feira, dia 8 de Fevereiro, ao final da tarde.

As provas poderão ser consultadas na 5ª-feira, dia 10 de Fevereiro, das 12:15 às 12:45, na sala 11.2.24.

notas do exame de recurso by acaetanoacaetano, 08 Feb 2011 10:19
Solução
acaetanoacaetano 01 Feb 2011 19:52
in discussion Hidden / Per page discussions » Quiz11

A afirmação é verdadeira, e foi essa também a escolha dos 14 votantes. Mas há para aí quem ache que continuidade implica diferenciabilidade, por aquilo que vi em testes/minitestes.

Solução by acaetanoacaetano, 01 Feb 2011 19:52
Solução
acaetanoacaetano 01 Feb 2011 19:46
in discussion Hidden / Per page discussions » Quiz10

Houve 8 votos contra e 5 a favor. A afirmação em si é falsa: a regra de Cauchy aplica-se apenas na presença de (certas) indeterminações, logo não se pode aplicar no segundo limite, o qual dá infinito sem sinal determinado.

Solução by acaetanoacaetano, 01 Feb 2011 19:46
Solução
acaetanoacaetano 01 Feb 2011 19:41
in discussion Hidden / Per page discussions » Quiz9

Todos acertaram aqui, isto é, a afirmação é falsa.

Solução by acaetanoacaetano, 01 Feb 2011 19:41
Solução
acaetanoacaetano 01 Feb 2011 19:37
in discussion Hidden / Per page discussions » Quiz7

O balanço desta votação é curioso: 12 a favor e 12 contra. A afirmação em si é verdadeira. A prova poderia ser, usando primeiro a hipótese $\delta < \delta_1$ e depois a hipótese (1),

(1)
\begin{align} \| x-a \| < \delta \Rightarrow \| x-a \| < \delta_1 \Rightarrow \| f(x) - b \| < \varepsilon. \end{align}

Como é que metade dos votantes conseguiu errar nesta questão é para mim um mistério insondável (mas claramente aponta para a necessidade de este tipo de raciocínio ser treinado mais cedo no percurso académico).

Solução by acaetanoacaetano, 01 Feb 2011 19:37
Solução
acaetanoacaetano 01 Feb 2011 19:27
in discussion Hidden / Per page discussions » Quiz6

Apenas um aluno dos 20 que votaram errou: a afirmação está, de facto, errada (a negação da hipótese não garante a negação da tese).

Solução by acaetanoacaetano, 01 Feb 2011 19:27
Solução
acaetanoacaetano 01 Feb 2011 19:24
in discussion Hidden / Per page discussions » Quiz5

Dos 24 alunos que participaram na votação, apenas 3 acertaram. A afirmação é, na verdade, falsa: o facto de os limites restrito a todas as rectas que passam pelo ponto de acumulação darem um mesmo valor não é garantia de que o limite (globalmente considerado) exista.

Solução by acaetanoacaetano, 01 Feb 2011 19:24
Re: quizzes
acaetanoacaetano 01 Feb 2011 19:18
in discussion Avisos e material vário / AVISOS » quizzes

Como o interesse dos quizzes se esgota essencialmente amanhã, com o exame de recurso, irei de seguida, no local próprio, proceder a uma indicação sumária do valor lógico de cada um. Apesar de vários não conterem votações em nº suficiente de acordo com o que foi mencionado anteriormente, deste modo pelo menos os alunos que reflectiram sobre eles e fizeram as suas escolhas, poderão comparar com a resposta "oficial" e, se necessário, ajustar a interiorização que fizeram dos conceitos em causa.

Re: quizzes by acaetanoacaetano, 01 Feb 2011 19:18

Isso está explicado em http://amiii.wikidot.com/2-3-integrais-curvilineos-parte-3, que inclui também um exemplo.

Re: ex. 7 e 8, folha 5 by acaetanoacaetano, 01 Feb 2011 18:55

Podia dizer-me como se resolvem exercícios como o 7 e o 8 onde não temos um ds mas sim dx, dy e dz?
Obrigada.

Re: ex. 7 e 8, folha 5 by Diana RodriguesDiana Rodrigues, 01 Feb 2011 12:27

Os alunos devem concentrar-se junto à sala 12.1.1.

Estarei no departamento na 2ª-feira, dia 31, das 14:00 até às 16:30, de modo que poderei tirar dúvidas presencialmente a alunos das TP1 e TP2 dentro desse horário. Poderei estender o horário mediante marcação prévia por e-mail por parte dos interessados. Se se conseguirem organizar de modo a não aparecerem individualmente, melhor, especialmente quando houver dúvidas comuns.

page 1123...next »
Unless otherwise stated, the content of this page is licensed under Creative Commons Attribution-Noncommercial-No Derivative Works 2.5 License.